LSAT and Law School Admissions Forum

Get expert LSAT preparation and law school admissions advice from PowerScore Test Preparation.

User avatar
 Dave Killoran
PowerScore Staff
  • PowerScore Staff
  • Posts: 5852
  • Joined: Mar 25, 2011
|
#43584
Complete Question Explanation
(The complete setup for this game can be found here: lsat/viewtopic.php?t=8007)

The correct answer choice is (B)

The correct answer choice is (B) W and Y. If a W and a Y were both selected, then G, H, and K would all be precluded from selection, since G cannot go with a Y, and H and K both need an X. This would leave only J and L as possible fish selections, which is unacceptable, because exactly three fish must be selected.

One effective method of attack on this question is to use the information from other questions and answer choices. For example, the information from question #12 can be used to instantly eliminate answer choice (A), and the information from question #14 can be used to instantly eliminate answer choice (E).
 Adam Tyson
PowerScore Staff
  • PowerScore Staff
  • Posts: 5153
  • Joined: Apr 14, 2011
|
#36884
User ValVal asked this question:
Hi, guys!

I also have a question about #16 in this game.
My set up is the following:
Y :dblline: G
H :dblline: G
H :arrow: K :arrow: X
J :arrow: W

So in Question 16, I got stuck between 2 answer choices: (A) and (B)
My reasoning is this:
If the hobbyist selects (A), which is W and X, he technically cannot select Y, because the plant group is full, nevertheless, the setup itself, as I see it, allows the possibility of choosing Y and thus violating the game scenario.

Almost identical reasoning behind answer choice (B): if he choses W and Y, he may or may not be prompted to chose X. So this option can equally violate or not to the game scenario.

So as you see, those answer choices both look pretty good to me.

Could you clarify this for me?

Thank you!
While ValVal asked about q16, it appears that we were actually looking at the answers to q17, so we'll take this opportunity to review that question here.

In this question, we need to find the group of plants that will not work together. This will mean that one of these pairs will force some rule violation - either some fish that cannot go together will have to (like G and H), or we will be unable to select enough fish, or a 3rd plant will get forced upon us. Here, unlike in q16, we have no local restriction about selecting Y, so it's just about reviewing these pairs of plants to see what else happens.

Answer A - we know this pair works just by looking at the answer to our list question, q12, in which W and X were the plants selected. No problem there!

Answer B - this is the correct answer. if we select W and Y, that means we do NO select either X or Z. With X not selected, we have to also remove K from contention, and that means further that we have to take H out of the running. That's all based on the contrapositive of the conditional chain we diagrammed of H :arrow: K :arrow: X. The selection of Y means that G cannot be selected either, from the first rule. Now we have only two fish left, J and L, and since the rules require three fish this combination of plants will not work.

Answer C - with W and Z selected, X and Y are out. As with B above, removing X knocks out H and K, leaving G, J and L as our fish. No conflict there with any of our rules, so it works and must be incorrect in this Except question. Don't forget that we are knocking out "good" answers and selecting the one that doesn't work!

Answer D - X and Y selected means W and Z are out. W out means J is out because of the contrapositive of the third rule. Y in means G is out per the first rule. That means our fish are H, K and L. Since Z is already in, that works, and is another wrong answer.

Answer E - X and Z selected mean that W and Y are out, and that again takes J out of the running for our fish selection, leaving GHKL to pick from. We cannot pick G and H together, but take either one of them out and the other three will be fine, either HKL or GKL. Either way, this combination works and this answer is incorrect.

Let's hope this clears things up!

Get the most out of your LSAT Prep Plus subscription.

Analyze and track your performance with our Testing and Analytics Package.